b33eazy
Thanks Received: 0
Vinny Gambini
Vinny Gambini
 
Posts: 3
Joined: July 11th, 2012
 
 
 

Q13 - Ethicist: Studies have documented

by b33eazy Fri Oct 26, 2012 4:33 pm

I don't understand why the answer is B. The problem with that is if I negate that that would strengthen the argument because he is saying that it is questionable to prescribe placebos. I selected C and I think I know why it may be wrong, it says indefensible and the author says questionable.
 
sukim764
Thanks Received: 3
Forum Guests
 
Posts: 27
Joined: March 09th, 2012
 
 
 

Re: Q13 - Ethicist: Studies have documented

by sukim764 Sat Nov 17, 2012 7:13 pm

b33eazy Wrote:I don't understand why the answer is B. The problem with that is if I negate that that would strengthen the argument because he is saying that it is questionable to prescribe placebos. I selected C and I think I know why it may be wrong, it says indefensible and the author says questionable.


Hey,
Let me see if I can provide you with a concise response to your post. No better place than the core: The ethicist concludes that administering placebos is ethically questionable. Why? Because a doctor might prescribe it just to get the patient a sense of satisfaction.

So in order for the ethicist to conclude whether or not administering a placebo is ethically justifiable, he needs to assume that motivation is a relevant factor. If you negate the correct answer choice, it does not strengthen but rather destroys the argument. If motivation isn't a relevant factor, then he wouldn't be able to conclude that such practice is ethically justifiable or not.

With regards to C, I agree with what you stated. However, I would like to add that C is wrong also because it speaks of medical treatment in general, while the argument is about administering. Lastly, even if we were to grant that medical treatment is synonymous to administering, it's definitely not necessary to assume that such medical treatment is ethically indefensible. In fact, if you were to negate it (that such practice is NOT ethically indefensible, in other words, defensible) the double negative would actually hurt the argument.

Let me know if anything that I said is unclear. I hope this is helpful.

Best,
Su

One quick note worth mentioning about answer choice C: The Manhattan Lsat guide says 'the core is not about doctors prescribing only a placebo... the "only" in the argument refers to the only reason that the doctor prescribes the placebo, not the only thing the doctor prescribes--he or she may be prescribing many other drugs along with that placebo.'
 
lorraineogan
Thanks Received: 0
Forum Guests
 
Posts: 10
Joined: August 23rd, 2012
 
 
 

Re: Q13 - Ethicist: Studies have documented

by lorraineogan Tue Apr 09, 2013 1:20 pm

I was caught between A and B.

I think I figured out why A is wrong. A says a patient's psychological satisfaction is not a consideration in administering medical treatment, whereas patient's satisfaction is relevant when administering medical treatment. Also, the "medical treatment" is not the same as the placebo effect, which is part of the conclusion.

Is this correct? Am I thinking like a good LSAT Studier?!!

I still don't quite understand why C is incorrect though. Maybe it's wrong because the author never assumes it is ethically *indefensible.* Rather, he states that is is ethically *questionable.* So I think that would be the reason why I would reject C. Is that good reasoning?
User avatar
 
rinagoldfield
Thanks Received: 308
Atticus Finch
Atticus Finch
 
Posts: 390
Joined: December 13th, 2011
 
This post thanked 1 time.
 
 

Re: Q13 - Ethicist: Studies have documented

by rinagoldfield Tue Apr 16, 2013 11:16 am

Great LSAT thinking, lorraineogan :-)

Here’s the argument core for this problem:

Doctors could administer placebos "just to give the patient satisfaction that something was being done"

-->

Administering placebos is ethically questionable

This argument assumes a lot about ethics. It implies that the doctor’s underlying reason for prescribing the placebo impacts the ethical-ness of that decision, but who cares WHY the doctor prescribes the placebo if the placebo benefits patients?

(An analogy: I bought my friend a birthday present just because she would’ve been offended if I didn’t buy her one. I didn’t actually care about her birthday or think she needed the gift. Is my purchase unethical? My motives aren’t pure, but is the action totally tainted?)

(A) contradicts the premise. The author thinks doctors might consider patient satisfaction when administering treatment.

(B) is correct. The argument assumes that the doctor’s reason for administering the placebo matters. This is what we’re looking for.

(C) is way too extreme and out of scope. First of all, the author believes placebos are ethically questionable, but never goes so far as to say they are indefensible. Also, the argument is about doctors administering placebos as a component of treatment, while this answer choice is about treatment plans that rely SOLELY on placebos.

(D) undermines rather than helps the argument.

(E) is out of scope. We’re talking about doctors here.
 
LSATeater
Thanks Received: 0
Vinny Gambini
Vinny Gambini
 
Posts: 13
Joined: July 22nd, 2013
 
 
 

Re: Q13 - Ethicist: Studies have documented

by LSATeater Wed Jul 24, 2013 9:05 pm

rinagoldfield Wrote:(A) contradicts the premise. The author thinks doctors might consider patient satisfaction when administering treatment.


Can you please elaborate on how A contradicts the premise? I felt the last sentence of the premise which stated "... a doctor might..." did not really reveal anything about actual considerations since it was, after all, a hypothetical.

I eliminated choice A through negation because even if a patient's psychological satisfaction WAS a consideration, it need not be the ONLY consideration (e.g., maybe a doctor felt badly sympathetic towards a worried relative who accompanied the patient and wanted to assure that relative the patient would be receiving [a non-placebo] treatment in which case the RELATIVE's psychological satisfaction would also be a factor).
User avatar
 
Mab6q
Thanks Received: 31
Atticus Finch
Atticus Finch
 
Posts: 290
Joined: June 30th, 2013
 
 
 

Re: Q13 - Ethicist: Studies have documented

by Mab6q Sat Aug 24, 2013 3:56 pm

Let me see if I can clear up why A is not right. A says " A patient's psychological consideration is not a consideration in administering medical treatment". At first glance, this looked attractive to me. But when I looked at it more carefully, I saw that it was basically the opposite of some of the background information stated in the stimulus, as some of the other posters have already stated. The stimulus says: "some doctors say that they administer placebos because medically effective treatments reinforce by the placebo effect sometimes helps patients recover faster than good treatment alone". So, from this, we know that the psychological consideration is accounted for by the doctors, so A is wrong to counter the truth of a premise.
"Just keep swimming"
 
foralexpark
Thanks Received: 2
Vinny Gambini
Vinny Gambini
 
Posts: 24
Joined: June 08th, 2013
 
 
trophy
First Responder
 

Re: Q13 - Ethicist: Studies have documented

by foralexpark Tue Sep 10, 2013 10:21 pm

I was also down to (a) and (b)

but heres my reason why I eliminated (a):

The conclusion is that..

Placebo is questionable because of ethics and its justification.

(A) does not even get close to becoming a "required" or "necessary" assumption that is needed to get this conclusion going... because it doesn't mention anything near ethics or justification or even morality or so forth...


(B) on the other hand, says that motivation for place can be relevant to ethics justification, and if we negate this and say:

"motivation for placebo is NOT relevant to ethics justification"

then the argument falls apart, because this negation clearly says who cares about ethics... placebo is the goodies.

let me know if I need to be corrected!
User avatar
 
WaltGrace1983
Thanks Received: 207
Atticus Finch
Atticus Finch
 
Posts: 837
Joined: March 30th, 2013
 
 
trophy
Most Thanked
trophy
Most Thankful
trophy
First Responder
 

Re: Q13 - Ethicist: Studies have documented

by WaltGrace1983 Thu Mar 13, 2014 1:01 pm

Wouldn't (D) and (E) both be wrong because they are talking about "justification" while the stimulus only talks about being "ethically questionable?"

(A) is it ethical or not?
(C) we aren't concerned with treatment that relies on placebo ALONE and we don't need to conclude being ethically INDEFENSIBLE
User avatar
 
ohthatpatrick
Thanks Received: 3806
Atticus Finch
Atticus Finch
 
Posts: 4661
Joined: April 01st, 2011
 
This post thanked 1 time.
 
 

Re: Q13 - Ethicist: Studies have documented

by ohthatpatrick Tue Mar 18, 2014 3:28 pm

If we extend your logic about (D) and (E) to answer choice (B), the correct answer, then (B) is also wrong simply because it talks about "ethical justification".

It would be dangerous for us to be insistent on seeing the words "ethically questionable", or else none of these answers would apply.

Saying that something is "ethically questionable" is akin to asking "hmm, is it ethically justified or not?"

So I don't think we can knock out (D) and (E) just for that.

I do agree with your sentiments about (A) and (C).
User avatar
 
WaltGrace1983
Thanks Received: 207
Atticus Finch
Atticus Finch
 
Posts: 837
Joined: March 30th, 2013
 
 
trophy
Most Thanked
trophy
Most Thankful
trophy
First Responder
 

Re: Q13 - Ethicist: Studies have documented

by WaltGrace1983 Tue Jun 24, 2014 12:39 pm

ohthatpatrick Wrote:If we extend your logic about (D) and (E) to answer choice (B), the correct answer, then (B) is also wrong simply because it talks about "ethical justification".

It would be dangerous for us to be insistent on seeing the words "ethically questionable", or else none of these answers would apply.

Saying that something is "ethically questionable" is akin to asking "hmm, is it ethically justified or not?"

So I don't think we can knock out (D) and (E) just for that.

I do agree with your sentiments about (A) and (C).


Doh :o ! Stupid mistake on my part! However, after returning to this question I have a bit to add (thanks for the analysis by the way, Patrick).

    Doctor might have prescribed it just to give the patient satisfaction
    →
    Administering placebos is ethnically questionable


The "just to" says that we are basically talking about the motivations of the doctor. He/she prescribed the medicine in order to do X.

    (A) The first problem with this is that it doesn't actually tie in the conclusion of the argument. While this is not an absolute 100% deal breaker for Necessary Assumption questions, it does make me suspicious. However, it also just blatantly goes against the premise. It is a premise de-booster if you will. Why? The premise says that some doctors have prescribed some placebos just to (motive) give a patient satisfaction. Clearly then, psychological satisfaction IS something to consider.

    (B) If it's not relevant, we have no reason to use it as the basis for claiming that administering placebos is "ethically questionable."

Now here's the fun part. I thought (C), (D), and (E) were REALLY close (and thus REALLY tempting). I am going to make some corrections to them and show how the LSAT could have dressed these up to make them correct. Humor me for a second, if you will.

(C) Medical treatment that relies on the placebo effect is ethically questionable.

I got rid of the "alone," because we have NO IDEA if this medical treatment is relying on ONLY the placebo effect (is this valid reason enough for eliminating C?). In addition, the more damning evidence that (C) sucks was that it talks about being "ethically indefensible." Hmmm, much too strong! Just because something is "questionable" doesn't mean it's "indefensible." In fact, I would argue that "questionable" is more like the exact opposite.

One thing I will add is that "relies on" is a bit strong too. For similar reasons why I got rid of "only," relies on might be a bit unsupported. What do you guys/girls think?


(D) The psychological effects produced by the placebo effect may not justify the deception involved in administering a placebo.

To me, this is a bit more like where I expected the LSAT to go. The argument must be assuming that the psychological effects do NOT necessarily justify the deception, otherwise it wouldn't say that the administration of placebos is "ethically questionable." Once again, we have to steer clear off to strong language. We couldn't just say that it "does not justify" because the conclusion was merely that it is "ethically questionable."

(E) Administering a placebo may not be ethically justified if that treatment is prescribed by a doctor

This is much less interesting. However, the main problem with (E) is that it failed the sufficient condition. We know that these placebos WERE prescribed by a doctor. We know nothing about if they were NOT prescribed by a doctor.

What do you think?
User avatar
 
ohthatpatrick
Thanks Received: 3806
Atticus Finch
Atticus Finch
 
Posts: 4661
Joined: April 01st, 2011
 
This post thanked 1 time.
 
 

Re: Q13 - Ethicist: Studies have documented

by ohthatpatrick Sat Jun 28, 2014 1:53 pm

I like this game.

The new (C) doesn't quite seem right to me because I don't think we were specifically discussing a treatment that RELIES ON or ONLY USES the placebo effect.

The 2nd sentence talks about using placebos IN ADDITION to medically effective treatment, because the placebo gives the treatment a boost.

So in that context, the treatment doesn't RELY on the placebo effect.

As you pointed out, the premise is really focused on the author's motivation. Here, it says that the doctor is prescribing the placebo JUST for the patient's satisfaction, not for some hopeful boost in efficacy.

So the author is not relying on the placebo effect at all in these situations. The doctor is just intending to appease the patient in some way.

We could say
(C) A medical treatment that is administered solely for the patient's satisfaction is ethically questionable.
 
cyt5015
Thanks Received: 6
Elle Woods
Elle Woods
 
Posts: 75
Joined: June 01st, 2013
 
 
 

Re: Q13 - Ethicist: Studies have documented

by cyt5015 Tue Dec 30, 2014 12:02 pm

Can any expert check if my reasoning to eliminate A is correct? I was trapped by A and mistakenly eliminated the correct answer B.
(A) wrong for two reasons: first, it is too strong by using IS NOT for the conclusion being just "ethically questionable" not "ethically unjustifiable". Negate A may wreck the whole argument but A is not a must be true assumption; second, we have to assume that if it is not a consideration in administrating medical treatment, then it is ethically questionable to do so.

(B) The reason I mistakenly eliminate it is because of "the motivation". If answer B gets rid of "the motivation" I will absolutely choose it. I thought the argument focuses on the ACTION of administering the placebo. Can anyone clarify this for me please? Thanks a lot!
In LSAT, DO we need to distinguish motivation of an action and action itself? in what circumstance?
User avatar
 
ohthatpatrick
Thanks Received: 3806
Atticus Finch
Atticus Finch
 
Posts: 4661
Joined: April 01st, 2011
 
This post thanked 1 time.
 
 

Re: Q13 - Ethicist: Studies have documented

by ohthatpatrick Fri Jan 02, 2015 1:03 am

You're super close in your rationale. Just a couple tweaks:

Negating (A) doesn't ruin the argument.

"A patient's psychological satisfaction IS a consideration in administering medical treatment".

Okay, does that mean that any medical treatment that caters to a patient's psychological satisfaction is ethically sound?

No, because we're just saying that patient satisfaction is a consideration.

Treatment decisions will naturally be concerned with many variables, and negating (A) just tells us that patient satisfaction is ONE of those variables.

That doesn't mean that we give a greenlight to any treatment that satisfies patients. So negating (A) doesn't actually hurt the argument at all.

The author isn't saying that placebos are ethically questionable because he thinks "Patient satisfaction is NEVER something we consider!"

He doesn't think it's ethically dubious to consider patient satisfaction; he thinks it's ethically dubious to ONLY consider patient satisfaction.

This is the segue to your concern about 'motivation' in (B).

There's only one premise in this argument.

CONC: administering P's is ethically questionable
(why?)
PREM: doc might be prescribing P just to give the patient satisfaction.

The bolded phrase is what tells you the premise is focused on the doctor's motivation, not her action.

If we say, "I went to the party just to return Sharon's scarf", we only have a fact about my motive for going to the party. We have no idea whether I actually was allowed to enter the party, whether I stayed, whether I got drunk and told Sharon what a fool I was to break up with her in the first place .... etc.

This is a stupid grammar point but when we use infinitives in English (to swim, to return, to give) we convey purpose.

If I say "Sharon drove through the puddle to splash me", we have no idea if her action actually resulted in splashing me. We only know about her intent.

When RC questions ask
"The author mentions ____ in order to"
"The author's reference to _____ serves primarily to"

... these are PURPOSE (motivation) questions, because all the answer choices are prefaced by "to".

In terms of your broader LSAT question, we ABSOLUTELY have to distinguish between actions and intentions quite a bit. (Normally we only know an action, and a trap answer baits us into assuming some intention)

In this case, our one and only premise is a statement about WHY a doctor prescribed a placebo, so motive/intent is fair game.

Hope this helps.
 
AnnaT620
Thanks Received: 0
Elle Woods
Elle Woods
 
Posts: 51
Joined: May 25th, 2020
 
 
 

Re: Q13 - Ethicist: Studies have documented

by AnnaT620 Sun Jul 19, 2020 7:17 am

I found the conclusion / argument core in this question ("administering placebos is nonetheless ethically questionable"), but I am not sure how to get from there to figuring out the gap in the argument / finding the necessary assumption here? Not sure I am clear on why is B correct here, whereas D is incorrect?

Many thanks!
Anna